Übergangs-Marathon Mathematik - Seite 2

Neue Frage »

Jayk Auf diesen Beitrag antworten »

Zitat:
Original von Guppi12
Zitat:
Aufgabe 18
Seien für alle (nicht notwendigerweise stetige) Funktionen und

Es gelte:

1. punktweise.
2. monoton wachsend für alle
3. stetig.

Zeige, dass gleichmäßig.



Zitat:
Lösung zu Aufgabe 18
Zeige zunächst: f ist monoton wachsend.

Dies folgt unmittelbar aus den Grenzwertsetzen: Seien , dann ist , da dies für alle Folgenglieder gilt und Grenzwertbildung Ordnungsrelationen erhält.

Sei nun . Sei k nacheinander 2, 3, ... Dann gibt es nach dem Zwischenwertsatz und wegen der Vollständigkeit von R Kontrollpunkte mit für , . Wegen der Monotonie von f gilt dann .

Nun sollen für festes k natürliche Zahlen (i wie oben) konstruiert werden, und zwar auf folgende Weise: Es gelte für alle . Wegen der punktweisen Konvergenz ist dies möglich. Es sei . Dann gilt wegen der Monotonieeigenschaften in jedem (definierten) Intervall :

.

Da diese Abschätzung auf jedem Teilintervall gilt, gilt sie auch auf ganz . Wegen ist damit gleichmäßige Konvergenz gezeigt.

Jayk Auf diesen Beitrag antworten »

Bei der entscheidenden Abschätzung fehlt natürlich ein .
Jayk Auf diesen Beitrag antworten »

korrigiert:

Zitat:
Lösung zu Aufgabe 18
Zeige zunächst: f ist monoton wachsend.

Dies folgt unmittelbar aus den Grenzwertsetzen: Seien , dann ist , da dies für alle Folgenglieder gilt und Grenzwertbildung Ordnungsrelationen erhält.

Sei nun . Sei k nacheinander 2, 3, ... Dann gibt es nach dem Zwischenwertsatz und wegen der Vollständigkeit von (in Gestalt der Supremums-/Infimumseigenschaft) Kontrollpunkte mit für , insbesondere. Wegen der Monotonie von f gilt dann .

Nun sollen für festes k natürliche Zahlen (i wie oben) konstruiert werden, und zwar auf folgende Weise: Es gelte für alle . Wegen der punktweisen Konvergenz ist dies möglich. Es sei . Dann gilt wegen der Monotonieeigenschaften in jedem (definierten) Intervall : . Es ist nämlich für : (wegen der Monotonie und der Definition des darauffolgenden Kontrollpunkts) und . Nun unterscheidet man die Fälle, in welchem Drittel des Intervalls sich befindet und erhält für die äußeren Drittel die Abschätzung gegen und für das innere Drittel gegen . Diese Abschätzung ist unabhängig von dem Teilintervall und gilt daher auf ganz . Nun ist aber , womit gleichmäßige Konvergenz gezeigt ist.


Zitat:
Aufgabe 19
Es sei zusammen mit einer Raumzeitstruktur: . Es sei eine affine Abbildung, also eine solche, die Geraden in Geraden überführt, von der außerdem bekannt ist, dass . Man zeige, dass es gibt, sodass mit den folgendermaßen definierten Elementartypen:


Jayk Auf diesen Beitrag antworten »

Hier nun hoffentlich fehlerfrei die neue Aufgabe 19:

Zitat:
Aufgabe 19
Sei bijektiv und differenzierbar, sei die dazugehörige Umkehrfunktion und es gelte . Sei . Finde eine Stammfunktion von auf durch Komposition von (d.h. erlaubt ist Addition, Subtraktion, Multiplikation, Division und Verkettung).

Hinweis zur Lösung: Die Differenzierbarkeit der Umkehrfunktion ist gesichert.
Leopold Auf diesen Beitrag antworten »

Zitat:
Lösung Aufgabe 19



Jayk Auf diesen Beitrag antworten »

Zitat:
Original von Leopold
Zitat:
Lösung Aufgabe 19





Ich wusste doch, dass die zu einfach ist... Augenzwinkern

Also dann, die nächste Aufgabe bitte.
 
 
Leopold Auf diesen Beitrag antworten »

Eine Rechenaufgabe.

Zitat:
Aufgabe 20



Dies sei als bekannt vorausgesetzt. Man verwende es, um



zu bestimmen.
RavenOnJ Auf diesen Beitrag antworten »

Zitat:
Lösung 20
Erst mal gilt das Additionstheorem:

Sodann mit partieller Integration



Der erste Ausdruck der rechten Seite der vorletzten Gleichung wird mit l'Hospital gleich Null (auch das Schreiben von als Potenzreihe ergibt dies sofort):


Leopold Auf diesen Beitrag antworten »

Stimmt. Freude

Dann gleich die nächste.

Zitat:
Aufgabe 21

Für eine ganze Zahl sei die zu nächstgelegene ganze Zahl. Man bestimme den Wert der Reihe

Tesserakt Auf diesen Beitrag antworten »

Zitat:
Aufgabe 21 Lösung
Wir teilen die Menge auf gemäß
.
Dann gilt .
Sei die Menge sowie und . Die Mengen und sind gewiss disjunkt und da für alle mit gilt, dass ist sowie . Folglich und , sodass . Da gemäß der Definition von und , ist also .
Wir wollen nun alle Elemente der Menge bestimmen. Sei . Da , muss offenbar gelten (Gleichheit ist ausgeschlossen, da andernfalls eine rationale nicht natürliche Zahl wäre, dies ist nicht möglich), gemäß der Definition von .
Leicht sieht man, dass die obige Ungleichung genau dann erfüllt ist, wenn gilt. Da , gilt also . Also und entsprechend .

Wir können folglich die Summe schreiben als

.

Es ergibt sich (Potenzgesetze, geometrische Reihe) sowie

Insgesamt ergibt sich so .

Neu anordnen ergibt .

Es gilt nun .
Fernerhin sowie

.

Es ergibt sich also letztendlich .

Da ergibt sich
.

Dies galt es zu bestimmen.

Tesserakt Auf diesen Beitrag antworten »

Mal etwas Einfaches zur Entspannung. Augenzwinkern

Zitat:
Aufgabe 22
Man zeige, dass der Bruch für alle vollständig gekürzt ist.
Leopold Auf diesen Beitrag antworten »

@ Tesserakt

Deine Lösung scheint mir richtig, wenn ich auch nicht jedes Detail durchgegangen bin (ist mir ein bißchen zu kompliziert). Meine Lösung geht ähnlich, ich gruppiere nur etwas anders. Ich verwende die Folge mit



Sie ist streng monoton wachsend (ihre Summanden sind es ja). Daher existiert zu jedem ganzzahligen ein mit



Wegen und folgt:



Das zeigt: .

Damit läßt sich der Reihenwert berechnen. Ich nehme die leicht verallgemeinerte Version von xb mit einem mit :





Leopold Auf diesen Beitrag antworten »

Zitat:
Lösungsidee Aufgabe 22

Leopold Auf diesen Beitrag antworten »

Und hier eine Aufgabe, die so oder ähnlich immer wieder mal herumgeistert. Ihre Lösung ist für Profi-Mathematiker natürlich keine Herausforderung. Die Aufgabe ist eher für Laien gedacht, und man kann damit seine Bekannten verblüffen. Auf den ersten Blick ist vielleicht sogar der Experte überrascht.

Zitat:
Aufgabe 23

Frisch geerntete Erdbeeren bestehen zu 90 % Gewichtsanteil aus Wasser.
Die Erdbeeren liegen einige Zeit in der prallen Sonne, so daß ihr Wasseranteil auf 80 % sinkt.
Wieviel Prozent ihres jetzigen Gewichts haben die Erdbeeren ursprünglich gewogen?
xb Auf diesen Beitrag antworten »

Zitat:
Lösung Aufgabe 23


xb Auf diesen Beitrag antworten »

Aufgabe 24

Eine Münze wird so lange geworfen bis entweder n mal Zahl
oder n mal Wappen erschienen ist

Zeige
die Wahrscheinlichkeit,dass man 2n-1 mal werfen muss ist
genauso groß wie die Wahrscheinlichkeit,dass man 2n-2 mal werfen muss
Huggy Auf diesen Beitrag antworten »

Zitat:
Lösung Aufgabe 24

Es sei , denn bei gilt die Behauptung nicht.

Bis zum Wurf sei kein Erfolg eingetreten. Man betrachte die möglichen Wurfkombinationen in den Würfen bis .
2 dieser Kombinationen, nämlich





führen im Wurf zum Erfolg, kommen also für einen möglichen Erfolg im Zug oder nicht mehr in Frage. Von den verbleibenden Kombinationen können nur diese 2





potentiell im Zug zum Erfolg führen. Sie tun dies jeweils mit Wahrschenlichkeit . Wenn sie zum Erfolg führen, kommen sie für einen Erfolg im Zug nicht mehr in Frage. Wenn sie nicht zum Erfolg, können sie offensichtlich auch im Zug nicht zum Erfolg führen.

Im Zug können daher potentiell nur folgende 4 Kombinationen zum Erfolg führen:









Sie tun dies jeweils mit Wahrscheinlichkeit .


Huggy Auf diesen Beitrag antworten »

Zitat:

Aufgabe 25

Auf einige Feldern eines Schachbretts werden Münzen gelegt, pro Feld höchstens eine Münze. Die Zahl der Münzen in jeder horizontalen Reihe sei ungerade. Die Zahl der Münzen in jeder vertikalen Linie sei ebenfalls ungerade.

Man beweise:
Die Zahl der Münzen auf den schwarzen Feldern ist gerade!
tmo Auf diesen Beitrag antworten »

Ist die das Schachbrett beschreibende Matrix, so gilt .
Huggy Auf diesen Beitrag antworten »

Zitat:
Original von tmo
Ist die das Schachbrett beschreibende Matrix, so gilt .

Kannst du diesen Beweis für einen in der Algebra nicht so Kundigen etwas erläutern. Ich hatte einen wesentlich elementareren Beweis im Auge.
Leopold Auf diesen Beitrag antworten »

Zitat:
Original von Huggy
Zitat:
Original von tmo
Ist die das Schachbrett beschreibende Matrix, so gilt .

Kannst du diesen Beweis für einen in der Algebra nicht so Kundigen etwas erläutern. Ich hatte einen wesentlich elementareren Beweis im Auge.


Ich versuche einmal, tmos Lösung auszuformulieren.
Das Schachbrett wird als achtreihige Matrix über , dem Körper mit zwei Elementen, aufgefaßt. Liegt eine Münze, schreibt man , liegt keine Münze, schreibt man . Es sei die Summe der Elemente der -ten Spalte. Nach Voraussetzung ist



Nun gilt



Also ist



Und jetzt zäumen wir das Pferd von der anderen Seite auf. Dazu sei die Summe der schwarzen und die Summe der weißen Felder der -ten Zeile. Nach Voraussetzung über die -te Zeilensumme ist



Nun gilt:



Also ist





Nach ist diese Summe aber , daher ist die Summe der Münzen auf den schwarzen Feldern gerade.

Ich hoffe, daß sich tmo dieses so oder ähnlich gedacht hat.
Huggy Auf diesen Beitrag antworten »

@Leopold
Vielen Dank für die ausführliche Erläuterung. Mein elementarer Beweis sieht so aus:

Man nummeriere die Linien und Reihen fortlaufend gemäß Bild. Die schwarzen Felder auf den Reihen mit ungerader Nummer werden mit A markiert. Die schwarzen Felder auf den Linien mit gerader Nummer werden mit B markiert. Damit sind alle schwarzen Felder markiert. Die weißen Felder auf den Reihen mit ungerader Nummer werden mit C markiert. Die übrigen weißen Felder bleiben unmarkiert.

[attach]35158[/attach]

Es seien a , b, c die Zahl der Münzen auf den mit A, B, C markierten Feldern. Dann ist die Zahl der Münzen auf den Reihen 1, 3, 5, 7. Dies ist eine Summe aus 4 ungeraden Zahlen, also gerade. Ebenso ist die Zahl der Münzen auf den Linien 2, 4, 6, 8 eine gerade Zahl. Dann ist auch



gerade. Dann muss aber gerade sein und das ist gerade die Zahl der Münzen auf den schwarzen Feldern.
tmo Auf diesen Beitrag antworten »

Ja, das was Leopold ausgeführt hat, ist eigentlich 1-zu-1 das, was ich mir gedacht habe.

Elementar könnte man das so formulieren:

Wenn man in einer Zeile die weißen Münzen zählt, so erhält man nach Voraussetzung genau die andere Parität, die man erhält, wenn man die schwarzen Münzen der selben Zeile zählt.

D.h. wenn wir in 4 Zeilen die schwarzen Münzen und in 4 Zeilen die weißen Münzen zählen, so erhalten wir die selbe Parität, wie wenn wir direkt in allen 8 Zeilen die Parität der schwarzen zählen.

Also erhalten wir die Lösung, wenn wir in den Zeilen 1,3,5,7 die schwarzen und in den Zeilen 2,4,6,8 die weißen Münzen zählen. Dies ist aber dasselbe wie alle Münzen in den Spalten 1,3,5,7 (bei mir ist oben links ein schwarzes Feld) zu zählen und nach Vorraussetzung ist dies die Summe vier ungerader Zahlen, also gerade.

PS: Das, was grün markiert ist, ist genau das, was der obige Ausdruck zählt. Daher die knappe Lösung. Mit Algebra hat das aber ausnahmsweise mal nichts zu tun Big Laugh
Leopold Auf diesen Beitrag antworten »

Hier noch eine Ergänzung zu Aufgabe 24.

Die Zufallsgröße , die die Anzahl der Würfe angebe, bis die eine oder andere Münzseite das -te Mal gefallen ist, besitzt die Verteilung



Man kann nachrechnen.

Und eine neue Aufgabe aus dem Bereich der Unterhaltungsmathematik.

Zitat:
Aufgabe 26

Das Staatsgefängnis in der Hauptstadt Quempu Lempu von Hontraruru ist überfüllt mit politischen Häftlingen. Zum 80. Geburtstag des Diktators Entepente ergeht eine Amnestie. Dem Wärter Simarar wird befohlen, zunächst jede Zellentüre aufzuschließen, dann vorne beginnend in derselben Reihenfolge jede zweite Tür wieder abzuschließen, danach jede dritte Tür zu- oder aufzuschließen, je nachdem, ob sie zuvor auf oder zu war. Und dann jede vierte Tür und jede fünfte und ... Natürlich muß Simarar aufpassen, daß ihm während des hektischen Auf- und Zusperrens kein Häftling entwischt, will er nicht selber um einen Kopf kürzer gemacht werden. Wessen Tür am Ende offen ist, dem winkt die Freiheit.
Wie großzügig erweist sich Entepente? Kommen überhaupt Häftlinge frei?


Präzisierung der Aufgabe: siehe hier.
tmo Auf diesen Beitrag antworten »

Die Anzahl der Statusänderungen der Tür mit der Nummer ist offenbar die Anzahl der Teiler der Zahl . Der entsprechende Häftling kommt also frei, wenn die Teileranzahl ungerade ist, also wenn eine Quadratzahl ist.

Jede Primzahl freizulassen, wäre großzügiger Big Laugh
RavenOnJ Auf diesen Beitrag antworten »

Ich gehe vom Zustand aus, in dem alle Türen geschlossen sind. Wie oft eine Tür auf- und zugeschlossen wird, ergibt sich aus der Zahl der Teiler der Zellennummer n, denn für jeden Teiler (inklusive 1) steht ein Schließvorgang. Diese Teilerzahl nenne ich . Ist die Tür am Ende offen, so muss sie die Zahl der Schließvorgänge ungerade sein, d.h. . Sei die Primfaktorzerlegung von n , dann ist die Zahl der Teiler . Es gilt nun


Ein Häftling kommt also dann frei, wenn die Primfaktorzerlegung seiner Zellennummer nur gerade Exponenten hat, d.h. wenn die Zellennummer eine Quadratzahl ist. Glück haben also die aus Zellen 1,4,9,16,25,36,...
tmo Auf diesen Beitrag antworten »

Eine mMn verblüffende Aussage:

Zitat:

Aufgabe 27

Wieder zu einem Schachbrett, aber dies mal zweiseitig unbegrenzt und unten links liegt eine einzige Münze:




Ein Zug besteht darin, dass man eine beliebige Münze entfernt, aber dafür zwei neue Münzen platziert, und zwar direkt über und direkt rechts von der alten Münze. Nach dem ersten Zug sieht das Feld also zwingend so aus:




Ein Zug ist nur gültig, wenn die beiden Felder, die dadurch neu besetzt werden, vorher frei waren.

Man zeige, dass man den durch Kreuze markierten Bereich nach endlich vielen Schritten nie komplett verlassen kann, d.h. nach jedem Zug liegt immer noch mind. eine Münze in diesem Bereich.



Leopold Auf diesen Beitrag antworten »

Zitat:
Original von tmo
Jede Primzahl freizulassen, wäre großzügiger Big Laugh

Es sei denn, es sitzen unendlich viele Häftlinge ein. Wollen wir aber von diesem unwahrscheinlichen Fall großzügig absehen ... Augenzwinkern

Ich habe die Aufgabe schon gelegentlich in der Schule gestellt. Für die Argumentation, daß nur die Quadratzahlen ungeradzahlig viele Teiler besitzen, braucht man nicht einmal die Primfaktorzerlegung. Jeder Teiler einer Zahl besitzt genau einen Komplementärteiler mit . Man kann daher die Teiler zu Paaren zusammenfassen, womit die Teilerzahl gerade ist, außer eben bei Quadratzahlen: . Hier ist sein eigener Komplementärteiler. Er bleibt solo.
Gast11022013 Auf diesen Beitrag antworten »

Ich hatte dich gerne als Lehrer gehabt.
RavenOnJ Auf diesen Beitrag antworten »

Trotz der schönen Lösungen von tmo/leopold und Huggy eine weitere (effektive dieselbe, nur mit anderen Worten):


Zitat:

Lösung 25
Seien (respektive ) die Zahlen der Münzen auf weißen (resp. schwarzen) Feldern in den (un-)geraden Zeilen.

Wegen der Voraussetzung "Zahl der Münzen in allen Spalten ungerade" gilt für die Gesamtzahl der Münzen in ungeraden Spalten:

und wegen der Voraussetzung "Zahl der Münzen in allen Zeilen ungerade" für die Gesamtzahl der Münzen in geraden Zeilen


Daraus folgt für die Zahl aller Münzen auf schwarzen Feldern:

Huggy Auf diesen Beitrag antworten »

Zitat:
Lösung Aufgabe 27
Das hat mich wirklich ziemlich verblüfft.

Kurzfassung:
Um die notwendigen Züge machen zu können, ergeben sich notwendige vorbereitende Züge. Um diese machen zu können, ergeben sich notwendige die vorbereitenden Züge vorbereitende Züge usw. Es ergibt sich ein infiniter Regress. Man kommt nie dazu, die ursprünglich notwendigen Züge zu Ende zu bringen.

Langfassung:
Mein Beweis besteht aus einer etwas holprigen vollständigen Induktion. Das Brett sei statt nach oben nach unten unbegrenzt. Züge werden durch die Koordinaten der weggenommenen Münze angegeben. Nach dem ersten Zug hat man folgende Konfiguration:

010
100
000


Die beiden Münzen befinden sich im kritischen Bereich, müssen also entfernt werden. Dazu sind folgende Züge notwendig:

(1, 2), (2, 2), (2, 1)

Danach hat man folgende Konfiguration:

0010
0110
1100
0000


Wichtig: Die Züge müssen nicht in dieser Reihenfolge erfolgen. Es können auch zwischendurch andere Züge erfolgen, sofern sie die obigen Züge nicht verhindern. Die nicht fetten Münzen können also schon entfernt sein. Die fetten Münzen müssen sich nicht gleichzeitig auf diesen Positionen befinden. Wenn die Münze (2,2) erscheint, kann sie aber erst entfernt werden, wenn vorher die Münzen (2,3) und (3,2) entfernt wurden.

Die fetten Münzen müssen entfernt werden, weil sie sich im kritischen Bereich befinden. Die beiden anderen Münzen zwar auch, sie werden aber für den Beweis nicht benötigt. Es werde jetzt generell ein Münzentrio auf den Positionen (i, i), (i, i + 1), (i + 1, i) betrachtet, das entfernt werden muss. Das obige Trio bildet den Induktionsanfang.

Um ein solches Trio zu entfernen, sind folgende Züge notwendig:

(i, i + 1), (i +1, i+1), (i +1, i), (i, i)

Man hat dann folgende Konfiguration:

00xxx
0011x
x111x
x11xx
xxxxx


Wichtig: siehe oben

Die Münzen auf den Positionen (i, i + 1) und (i+1, i) müssen noch immer entfernt werden. Dazu muss vorher das fette Trio auf den Positionen (i + 1 , i + 1), (i +1, i + 2), (i + 2, i + 1) entfernt werden. Das ist der Schluss von i auf i + 1.
Huggy Auf diesen Beitrag antworten »

Zitat:
Aufgabe 28

Zwei Teams A und B tragen einen Wettkampf aus. Team A gewinnt ein einzelnes Spiel mit Wahrscheinlichkeit p. Mit Wahrscheinlichkeit r geht das Spiel unentschieden aus. Mit Wahrscheinlichkeit q = 1 - p - r gewinnt Team B das Spiel. Den Wettkampf gewinnt das Team, das zuerst 2 Siege mehr hat als das andere Team.

Mit welcher Wahrscheinlichkeit gewinnt Team A den Wettkampf?
tmo Auf diesen Beitrag antworten »

Leopold hat zwar schon eine (offenbar richtige) Lösung angekündigt, taucht aber seitdem nicht mehr auf, deswegen mache ich hier mal weiter:

Der Trick ist, die Unentschieden einfach mal zu vergessen:
Die Wahrscheinlichkeit, dass der nächste Sieg Team A gehört, ist und entsprechend für Team B. Ersetzen wir nötigenfalls also durch , so können wir o.B.d.A davon ausgehen, dass gilt, d.h. ein Unentschieden ist nicht mehr möglich.

Dies hat zur Folge, dass ein Ende des Wettkampfes nur nach einer geraden Anzahl an Spielen möglich ist. Wir fassen nun immer 2 Spiele zusammen. In 2 Spielen gibt es die 3 Möglichkeiten Team A gewinnt beide, Team B gewinnt beide, Beide gewinnen einen Spiel mit Wahrscheinlichkeit .

Offenbar gewinnt Team A den Wettkampf genau dann nach 2n Spielen, wenn das Ereignis Beide gewinnen ein Spiel zuvor n-1 mal hintereinander eingetreten ist und danach das Ereignis Team A gewinnt beide eintritt. Dies entspricht einer Wahrscheinlichkeit von .

Die Gesamtwahrscheinlichkeit ergibt sich also zu

.
tmo Auf diesen Beitrag antworten »

Mal ein Klassiker:

Zitat:

Aufgabe 29:

Man zeige, dass für eine Polynomfunktion vom Grad mit Leitkoeffizient ist.
Leopold Auf diesen Beitrag antworten »

Ich habe Aufgabe 28 so gelöst.

bezeichne das Ereignis, daß Team A das Turnier gewinnt. Ferner sei .

Für gibt es die folgenden vier disjunkten Fälle, die zusammen ganz ausmachen:


i) Wenn Team A das erste Spiel gewinnt, könnte es nach null, ein, zwei, drei usw. Unentschieden das zweite Spiel gewinnen (Wahrscheinlichkeiten usw.).
Zusammen macht das:

ii) Wenn Team A das erste Spiel verliert und nach null, ein, zwei, drei usw. Unentschieden Team B gewinnt (Wahrscheinlichkeiten usw.), ist man wieder in der Ausgangssituation (Wahrscheinlichkeit ).
Zusammen macht das:

iii) Wenn das erste Spiel unentschieden ist (Wahrscheinlichkeit ), ist man wieder in der Ausgangssituation (wahrscheinlichkeit ).
Zusammen macht das:

iv) Wenn B das erste Spiel gewinnt und nach null, ein, zwei, drei usw. Unentschieden A das nächste Spiel gewinnt, ist man wieder in der Ausgangssituation, wie Fall ii).


Die Summe dieser Wahrscheinlichkeiten ergibt , also



Wenn man nach auflöst und eliminiert, erhält man

Leopold Auf diesen Beitrag antworten »

zu Aufgabe 29:

gleich nach dem Urknall des MatheBoards
tmo Auf diesen Beitrag antworten »

Dann werte ich das mal als Lösung. Ist meiner Meinung nach eine Sache, die man als Mathematikstudent mal gesehen/gemacht haben sollte.
Leopold Auf diesen Beitrag antworten »

Zitat:
Aufgabe 30

Zwei reelle Folgen werden rekursiv definiert durch





Man zeige, daß die Folgen konvergieren, und bestimme ihren Grenzwert.
tmo Auf diesen Beitrag antworten »

Es wundert mich, dass diese Aufgabe so wenig Anklang findet. Wenn man die ersten Folgenglieder hinschreibt, sieht man ja, was passiert.

Und zwar definieren wir eine neue Folge . Diese genügt nun der Rekursion .

Diese Folge hat folgende explizite Form: , wobei beide durch die Rekursion , aber unterschiedlichen Startwerten gegeben sind.

Die Startwerte sind klar und der Rest ist eine leichte Induktion.

Diese Differenzengleichung kann man leicht lösen und findet , sowie .

Alles zusammen ergibt .
Leopold Auf diesen Beitrag antworten »

Womit auch diese Aufgabe gelöst wäre!
Auch wenn die Beteiligung im Moment gering ist, stelle ich gleich die nächste Aufgabe.

Das Heronsche Verfahren zur Berechnung der Quadratwurzel einer reellen Zahl dürfte bekannt sein. Man setzt rekursiv:



Wenn man mit einem beliebigen reellen Startwert beginnt, so konvergiert die Folge gegen .

Wie ist es nun, wenn man in der Rekursionsformel die gewöhnliche Division durch die Ganzzahldivision ersetzt und die Sache über den ganzen Zahlen betrachtet?


Zitat:
Aufgabe 31

Für zwei ganze Zahlen bezeichne die Ganzzahldivision, also .

Man beginnt mit einer ganzen Zahl und setzt rekursiv fest:



Dabei sei auch eine ganze Zahl (und natürlich soll stärker als binden).

Man untersuche das Verhalten der Folge für .
Neue Frage »
Antworten »



Verwandte Themen

Die Beliebtesten »
Die Größten »
Die Neuesten »